ChaseDream
搜索
返回列表 发新帖
楼主: sdcar2010
打印 上一主题 下一主题

SDCAR2010【逻辑入门】(八)Assumptions

[精华]   [复制链接]
81#
发表于 2011-10-3 19:30:48 | 只看该作者
A question  again
for Q1,my idea is: a jump exits between  ticketed for exceeding the speed limit and more likely to exceed the speed limit ,while the (b) links the former concept with the latter one, as a result ,the (b) is more likely the corret answer than the rest.
is that right?
Besides, after reading your series, I can got correct answers more frequently, however , I have to spend a lot of time gaining them for I have to read and understand the stimulus and the options carefully. So what should I do for my pace now? How can I practise for both the high rate of correction and ideal pace? What is the ideal speed of one CR question for the test?
Thanks
82#
 楼主| 发表于 2011-10-3 22:08:56 | 只看该作者
Any gap or jump in the logic chain can be filled either with a sufficient assumption or a necessary one. So finding a gap definitely helps answer an assumption question. In that sense, your approach is a good one.

First, read the passage and find the premises and the conclusion. Then read all answer choices. Once locating all POSSIBLLY correct answer choices after eliminating those unlikely ones, your next task is to find the best among all the remaining POSSIBLE choices. This would be my approach to CR questions.

As to the pace, it varies. On average, a good test-taker can solve a CR question within 1 minute and 15 seconds. For easy ones, 30-second is enough. For tough ones, 2-3 minutes are not uncommon.

One aspect of any standard test is to frustrate the test-taker and see if the frustration affects his/her performance. Another is to test his or her judgment to pick and choose. To spend 5 minutes on a single question is insane. Better work on a solvable question in stead of wasting time on a dead-end, futile attempt.

A question  again
for Q1,my idea is: a jump exits between  ticketed for exceeding the speed limit and more likely to exceed the speed limit ,while the (b) links the former concept with the latter one, as a result ,the (b) is more likely the corret answer than the rest.
is that right?
Besides, after reading your series, I can got correct answers more frequently, however , I have to spend a lot of time gaining them for I have to read and understand the stimulus and the options carefully. So what should I do for my pace now? How can I practise for both the high rate of correction and ideal pace? What is the ideal speed of one CR question for the test?
Thanks
-- by 会员 suntree846 (2011/10/3 19:30:48)


83#
发表于 2011-10-3 22:31:34 | 只看该作者
受益良多哟~谢谢!
期待你的flaw(2)~
84#
发表于 2011-10-4 22:44:14 | 只看该作者

Q2

Hi. dear SDCAR2010, for question2, I choose answer A, and still can't figure out what's wrong. Would you please explain it to me? Thank you very much!!!
85#
 楼主| 发表于 2011-10-5 00:45:17 | 只看该作者
2. A researcher discovered that people who have low levels of immune-system activity tend to score much lower on tests of mental health than do people with normal or high immune-system activity. The researcher concluded from this experiment that the immune system protects against mental illness as well as against physical disease.
The researcher's conclusion depends on which of the following assumptions?
(A) High immune-system activity protects against mental illness better than normal immune system activity does.
(B) Mental illness is similar to physical disease in its effects on body systems.
(C) People with high immune-system activity cannot develop mental illness.
(D) Mental illness does not cause people's immune system activity to decrease.
(E) Psychological treatment of mental illness is not as effective as is medical treatment.

If you negate (D), you have:
Mental illness DOES cause people's immune system activity to decrease.
If that is true, then the author’s conclusion that the immune system protects against mental illness is wrong. So if you negate (D), the stimulus falls apart. Thus, (D) is the correct answer.


In A), there are two new concepts: high-immune system activity and normal-immune system activity. Both are NEW information, which are almost ALWAYS wrong for a NECESSARY assumption question simply because the new information is not required for the original passage/argument to hold.


Hi. dear SDCAR2010, for question2, I choose answer A, and still can't figure out what's wrong. Would you please explain it to me? Thank you very much!!!
-- by 会员 QUARTZ7 (2011/10/4 22:44:14)

86#
发表于 2011-10-5 15:39:48 | 只看该作者
2. A researcher discovered that people who have low levels of immune-system activity tend to score much lower on tests of mental health than do people with normal or high immune-system activity. The researcher concluded from this experiment that the immune system protects against mental illness as well as against physical disease.
The researcher's conclusion depends on which of the following assumptions?
(A) High immune-system activity protects against mental illness better than normal immune system activity does.
(B) Mental illness is similar to physical disease in its effects on body systems.
(C) People with high immune-system activity cannot develop mental illness.
(D) Mental illness does not cause people's immune system activity to decrease.
(E) Psychological treatment of mental illness is not as effective as is medical treatment.

If you negate (D), you have:
Mental illness DOES cause people's immune system activity to decrease.
If that is true, then the author’s conclusion that the immune system protects against mental illness is wrong. So if you negate (D), the stimulus falls apart. Thus, (D) is the correct answer.


In A), there are two new concepts: high-immune system activity and normal-immune system activity. Both are NEW information, which are almost ALWAYS wrong for a NECESSARY assumption question simply because the new information is not required for the original passage/argument to hold.



-- by 会员 sdcar2010 (2011/10/5 0:45:17)



Thanks!! But I'm still puzzled. Are high-immune system activity and normal immune system activity new information? The passage has mentioned them  "people with normal or high immune-system activity " .
Or, can I say that A is wrong, because if I negate A:
High immune system activity protects against mental illness worse than normal immune system activity does, it doesn't mean that immunne system can't protect against mental illness.
Hope for your explanation. Thank you very much!

87#
发表于 2011-10-5 16:20:34 | 只看该作者
Besides, dear SCARD2010,  I don't understand clearly what's the importance of finding the conclusion and premises in the type of ASSUMPTION questions, as in the BF ones. Are the main uses finding the jumps and negating the answers?  Thanks!!!
88#
 楼主| 发表于 2011-10-5 20:36:08 | 只看该作者
Quart27, you are right about the high- normal-immune system NOT being new info. My bad.

However, both were treated as having the same effect when compared against low-immune system. The conclusion focused on the difference between low- and [high-, normal]. Since A focus on the difference between high- and normal-, it is not necessary for the the main conclusion to hold.
89#
发表于 2011-10-7 09:25:07 | 只看该作者
Quart27, you are right about the high- normal-immune system NOT being new info. My bad.

However, both were treated as having the same effect when compared against low-immune system. The conclusion focused on the difference between low- and [high-, normal]. Since A focus on the difference between high- and normal-, it is not necessary for the the main conclusion to hold.
-- by 会员 sdcar2010 (2011/10/5 20:36:08)



I'm really appreciated your help!
90#
发表于 2011-10-7 21:13:14 | 只看该作者
Hi, NN, that is very kind of u.
I have read the post, but i do not know why do we seperate sufficient assumptions from necessary assumptions?
I hardly distinguish them!!
您需要登录后才可以回帖 登录 | 立即注册

Mark一下! 看一下! 顶楼主! 感谢分享! 快速回复:

手机版|ChaseDream|GMT+8, 2024-11-22 08:31
京公网安备11010202008513号 京ICP证101109号 京ICP备12012021号

ChaseDream 论坛

© 2003-2023 ChaseDream.com. All Rights Reserved.

返回顶部